Question

When frequency division multiplexing (FDM) is used to aggregate several modulated channels together, no separation between...

When frequency division multiplexing (FDM) is used to aggregate several modulated channels together, no separation between channels (i.e., use of guard channels) is required.

True

False

Regarding amplitude modulation (AM), if the AM index, µ, is greater than 1, then the modulated passband signal will begin to show signs of signal distortion.

True

False

Given the following AM modulated signal, determine the occupied frequency bandwidth (note: assume double-sideband large carrier (DSB-LC):

s(t) = 4[1+0.95 cos(2π3000t)] cos(2π100E9t)

a. Frequency bandwidth = 3000 Hz

b. Frequency bandwidth = 6000 Hz

c. Frequency bandwidth = 100E9 Hz

d. Frequency bandwidth = 200E9 Hz

Given the following message and carrier equations, determine the AM modulated signal equation: m(t)=3cos(2π4kHz*t), c(t)=2cos(2π3MHz*t)

a. s(t)=3[1 + 0.67cos(2π4000t)]cos(2π3E6t)

b. s(t)=2[1 + 1.5cos(2π4000t)]cos(2π3E6t)

c. s(t)=3[1 + 0.67cos(2π3E6t)]cos(2π4kHz*t)

d. s(t)=2[1 + 1.5cos(2π3MHz*t)]cos(2π4000t)

Given the following FM modulated signal equation, determine the frequency bandwidth. s(t)=2cos(2π1E9t + 4sin(2π1E3t))

a. 20kHz

b. 10kHz

c. 1kHz

d. 106 kHz

Given the following message, m(t), carrier, c(t), and Kp=0.1(rad/v), determine the PM modulated signal equation and PM index. m(t)=2cos(2π4000t), c(t)=4cos(2π8MHzt).

a. s(t)=4cos(2π4E6t + 0.75cos(2π3500t)), µPM=0.75

b. s(t)=4cos(2π8E6t + 0.2cos(2π4E3t)), µPM=0.2

c. s(t)=3cos(2π4E6t + 0.75cos(2π3500t)), µPM=0.75

d. s(t)=4cos(2π4000t + 0.2cos(2π8E6t)), µPM=0.2

Given the following PM modulated signal equation, determine the original message equation, m(t). s(t)=3.2cos(2π2E9t + 0.99cos(2π5000t)), Kp=0.33 (Hz/v)

a. m(t)= 4.4cos(2π5000t)

b. m(t)= 4.4cos(2π2E9t)

c. m(t)= 3cos(2π5000t)

d. m(t)= 3.2cos(2π2E9t)

Given the following PM modulated signal equation, determine the frequency bandwidth. s(t)=3cos(2π13E9t + 2cos(2π13E3t))

a. 22kHz

b. 40kHz

c. 78 kHz

d. 116 kHz

0 0
Add a comment Improve this question Transcribed image text
Answer #1

I ddealy or theoretically no gevind band is required but for this ideal filter is required But sideal filter are difficult tomt) = 3 ox (2 484) - Am Loo at tat M= Am = 3 =1.5 (4= 2 Cox (2113mt) - Ac cos anfct AC 84) Al II + M Colle en font] cop can fMA= 3 log (205000t) ③ part 84)= B Los [QH 13 € 97 +2 Lox (2113€ 37)] SA= Ac Cop[anfit + ß cop (at Int)] Compan, ß-2, fm 1363

Add a comment
Know the answer?
Add Answer to:
When frequency division multiplexing (FDM) is used to aggregate several modulated channels together, no separation between...
Your Answer:

Post as a guest

Your Name:

What's your source?

Earn Coins

Coins can be redeemed for fabulous gifts.

Not the answer you're looking for? Ask your own homework help question. Our experts will answer your question WITHIN MINUTES for Free.
Similar Homework Help Questions
  • Given the following message and carrier equations, determine the AM modulated signal equation: m(t)=4cos(2π4kHz*t), c(t)=4cos(2π3GHz*t) a....

    Given the following message and carrier equations, determine the AM modulated signal equation: m(t)=4cos(2π4kHz*t), c(t)=4cos(2π3GHz*t) a. s(t)=2[1 + 0.67cos(2π4000t)]cos(2π3GHz*t) b. s(t)=2[1 + 0.67cos(2π3E9t)]cos(2π4kHz*t) c. s(t)=4[1 + 1cos(2π4000t)]cos(2π3E9t) d. s(t)=4[1 + 1.5cos(2π3GHz*t)]cos(2π4000t)

  • below is used to modulate a carrier to generate the AM signal yt) (Am()) cos(1000t) The periodic signal m() shown a) The power efficiency of the system is measured to be 1 () The type of AM modula...

    below is used to modulate a carrier to generate the AM signal yt) (Am()) cos(1000t) The periodic signal m() shown a) The power efficiency of the system is measured to be 1 () The type of AM modulation, i.e. SSB, DSBSC, (i) The constant A. (ili) The modulation index 1/13. Determine DSB with carrier enit -3 b) An angle modulated signal is given by d(t-2 cos (2n1061+ sin 2 1000). Find i) The carrier frequency. ii) The baseband signal bandwidth....

  • 5. Consider the following message signal: -2 Assume that the carrier frequency is f are kr 105, k...

    5. Consider the following message signal: -2 Assume that the carrier frequency is f are kr 105, kp 25 respectively. 108 Hz, and frequency and phase deviation constants Find the maximum and minimum frequency deviation for FM, and sketch the FM wave for a duration of 103 seconds shown in the above figure (5 points). a. b. Find the maximum and minimum frequency deviation for PM, sketch the PM wave for a duration of 103 seconds shown in the above...

  • 5. Consider the following message signal: m(t) -2 .3 10 Assume that the carrier frequency is f ar...

    5. Consider the following message signal: m(t) -2 .3 10 Assume that the carrier frequency is f are kf = 105, kp-25 respectively. 108 Hz, and frequency and phase deviation constants Find the maximum and minimum frequency deviation for FM, and sketch the FM wave for a duration of 103 seconds shown in the above figure (5 points). a. b. Find the maximum and minimum frequency deviation for PM, sketch the PM wave for a duration of 103 seconds shown...

  • Question 4: (20 points) An FM signal is modulated with message m(t)Am cos(2Tmt). The measured amp...

    Question 4: (20 points) An FM signal is modulated with message m(t)Am cos(2Tmt). The measured amplitude spectrum, SPM(f), is shown below: Amplitude spectrum of the frequency modulated signal 2 -2 5000 4000 3000 2000 -1000 1000 2000 3000 4000 5000 frequency f [Hz] a) What is the carrier frequency fe? b) What is the message frequency fm c) What is the FM modulation index Bf? Is it a narrowband or a wideband signal? d) Using Carson's rule, estimate the transmission...

  • Communication systems d) The angle modulated signal is presented as = 1008 (2*10*4 +0.5sin(10*x+)) 1. Find...

    Communication systems d) The angle modulated signal is presented as = 1008 (2*10*4 +0.5sin(10*x+)) 1. Find and determine the frequency modulated signal x(C) with k, = 5. (5 marks) II. Calculate and find out the message signal m(t). (5 marks) e) A 40 MHz carrier is frequency-modulated by a sinusoidal signal with mascimum frequency deviation of 500 KHz. I. Find and calculate the approximate bandwidth of the FM signal the frequency of the message signal is 1 kHz. (5 marks)...

  • 2. [20 points] The carrier c(t-Acce(2106 t) is frequency modulated by the sinusoid signal m(t) 2cos...

    2. [20 points] The carrier c(t-Acce(2106 t) is frequency modulated by the sinusoid signal m(t) 2cos (2000mt). The deviation constant is kr 3000 Hz/V Determine the bandwidth of the modulated signal using Carson's rule a. Sketch the magnitude of the spectrum of the modulated signal (plot only those frequency components that lie within the bandwidth derived in part a) with detailed information such as the areas of delta function determined by the Bessel function, frequency separation between each spectrum (note...

  • Find the instantaneous frequency of the next signal at t = 4. (a) x(t) = 10...

    Find the instantaneous frequency of the next signal at t = 4. (a) x(t) = 10 cos[20xt+10 sin(t/4)) (b) x(t) = 262719) () X(t)=10[cos(2nt) cos(atº/4) + sin(24t) sin( nt/ 4)] Suppose that the information signal is as shown in the following figure. When each modulating this signal using 100KHz as the carrier frequency, find the frequency shift and phase shift when FM is modulated with 5,=10. What is the maximum frequency shift and maximum phase shift? 0.5+ -0.5 ost Suppose...

  • 5) given the following PM modulated signal equation, determine the original message equation, m(t) . s(t)=...

    5) given the following PM modulated signal equation, determine the original message equation, m(t) . s(t)= 3.5cos(2π2E9t + 0.5cos(2π6000t)), Kp=0.1 (hz/v) a) m(t)=0.16cos(2π6000t) b) m(t) = 5cos(2π6E3t) c) m(t) = 5cos(2π2E9t) d) m(t) = 0.16cos(2π2.4E9t) 6) given the following message and carrier waves. Determine the AM signal equation and (DSB-LC) bandwidth: m(t) =2sin(2π10KHz*t). c(t) =3sin(2π2.4E9Hz*t) a) 5[1+0.6sin(200πt)]sin(2π10E9t), BW=10kHz b) 3[1+0.67 sin(2π10E3t)]sin(2π2.4E9t), BW=20kHz c) 3[1+0.6sin(2.4GHz*πt)]sin(2π10E3Hz*t), BW=20kHz d) 3[1+1.67 sin(2π50t)]sin(2π10E9tt), BW=10kHz 7) amplitude used in a communication path will increase both signal...

  • Problem 1: An angle modulated waveform is given as s(t)=A_cos (1000 at +8071 +3204) where 0.=10001...

    Problem 1: An angle modulated waveform is given as s(t)=A_cos (1000 at +8071 +3204) where 0.=10001 , max|m(t)=3 , and m(t)=0,+<0. In a previous ICE, we determined that if the signal is phase modulated (PM), the phase sensitivity is k= 4001 - 133.31 , the message signal is m(t)=1 12 and the frequency deviation is Am=-80 tn 5-55.51 . Similarly, if the signal is frequency modulated (FM), the frequency sensitivity is kr , the message signal is m(t)==3(3), and the...

ADVERTISEMENT
Free Homework Help App
Download From Google Play
Scan Your Homework
to Get Instant Free Answers
Need Online Homework Help?
Ask a Question
Get Answers For Free
Most questions answered within 3 hours.
ADVERTISEMENT
ADVERTISEMENT
ADVERTISEMENT